inequality

This topic has expert replies
Senior | Next Rank: 100 Posts
Posts: 72
Joined: Tue Nov 05, 2013 4:35 pm
Followed by:4 members

inequality

by Ankitaverma » Mon Dec 09, 2013 10:19 am
Is x > y ?
(1) ax > ay
(2) a^2x > a^2y

Q/A-B can someone explain

User avatar
GMAT Instructor
Posts: 1052
Joined: Fri May 21, 2010 1:30 am
Thanked: 335 times
Followed by:98 members

by Patrick_GMATFix » Mon Dec 09, 2013 12:50 pm
Unless I'm missing something, there is a problem with the question as you posted it. As written, the statements do not give us enough information, even when combined so the official answer should not be what you indicated.

Statement 1
Simplify (1) by dividing both sides by a. If a is positive, the inequality is maintained, and (1) becomes x>y. On the other hand, if a is negative, dividing by a will flip the sign, and (1) becomes x<y.
Because we can get conflicting results, statement (1) is NOT SUFFICIENT

Statement 2
We can think of a^2x as (a^2)^x. Likewise, a^2y = (a^2)^y. Thus, (2) can be rewritten as (a^2)^x > (a^2)^y. Now, just think of a^2 as an unknown positive value. If this value is greater than 1, for example if a^2 = 4, then (a^2)^x > (a^2)^y would mean that x>y. However, if this value is less than 1, for example if a^2=1/4, then (a^2)^x > (a^2)^y would mean that x<y (because as the exponent of a proper fraction increases, the result only gets smaller; for example, x=1, y=2 would work).
Because we can get conflicting results, statement (2) is NOT SUFFICIENT

Merge Statements
The quick way: we determined from (1) that x>y if a is positive, but x<y if a is negative. So for the statements together to be sufficient, (2) should give us a way to determine the sign of a. Since (2) gives us info on a^2, there is no way to isolate the sign of a. Thus the statements together are not sufficient..

The other quick way: we determined from (2) that x>y if a^2>1 (meaning if a>1 or <-1), but x<y if a^2<1 (meaning a is between -1 and 1). So for the statements together to be sufficient, (1) should give us a way to determine whether a falls between -1 and 1. Since (1) doesn't put any restriction on the value of a, we cannot determine this. Thus the statements together are not sufficient.

We can prove that the statements are not sufficient together by plugging in values that agree with both statements but still yield conflicting answers.

If a=2, x=2, y=1 both statements are respected, and x>y
If a=-1/2, x=1, y=2 both statements are respected, and x<y
We have conflicting results. The answer is E

If you correctly copied the question, beware of its source.

Hope this helps. Read my signature below for more
-Patrick
  • Ask me about tutoring.

GMAT/MBA Expert

User avatar
GMAT Instructor
Posts: 16207
Joined: Mon Dec 08, 2008 6:26 pm
Location: Vancouver, BC
Thanked: 5254 times
Followed by:1268 members
GMAT Score:770

by Brent@GMATPrepNow » Mon Dec 09, 2013 1:05 pm
Ankitaverma wrote:Is x > y ?

(1) ax > ay
(2) a²x > a²y
Based on the official answer (B), I believe the question should be worded as I have it above.

Target question: Is x > y ?

Statement 1: ax > ay
Some students will divide both sides by a and incorrectly conclude that x > y.
However, before we divide by a variable, we must ensure that the variable is EITHER positive OR negative, because if we divide by a negative value, we must reverse the direction of the inequality, and if we divide by a positive value, the direction of the inequality stays the same.

To see what I mean, consider these values of a, x and y that satisfy the given condition:
Case a: a = 1, x = 3 and y = 2, in which case x > y
Case b: a = -1, x = 2 and y = 3, in which case x < y
Since we cannot answer the target question with certainty, statement 1 is NOT SUFFICIENT

Statement 2: a²x > a²y
First recognize that a² > 0 for ANY value of a
Also recognize that a ≠ 0, since the inequality would not hold true of a = 0.
This means that a² MUST BE POSITIVE
Since we can be certain that a² is positive, we can divide both sides of the inequality by a² to get x > y
In other words, it must be true that x > y
Since we can answer the target question with certainty, statement 2 is SUFFICIENT

Answer = B

Cheers,
Brent
Brent Hanneson - Creator of GMATPrepNow.com
Image

User avatar
GMAT Instructor
Posts: 1052
Joined: Fri May 21, 2010 1:30 am
Thanked: 335 times
Followed by:98 members

by Patrick_GMATFix » Mon Dec 09, 2013 1:38 pm
Brent@GMATPrepNow wrote:I believe the question should be worded as I have it above.

Makes sense.
  • Ask me about tutoring.

User avatar
Legendary Member
Posts: 1556
Joined: Tue Aug 14, 2012 11:18 pm
Thanked: 448 times
Followed by:34 members
GMAT Score:650

by theCodeToGMAT » Mon Dec 09, 2013 10:27 pm
To find: x > y

Statement 1:
ax > ay
if a > 0 ==> x > y
if a < 0 ==> x < y
INSUFFICIENT

Statement 2:
a^2x > a^2y
It doesn't matter now whether what sign does "a" has.. as its (a)^2
So, we can strike-out a^2 from both sides
x > y
SUFFICIENT

Answer [spoiler]{B}[/spoiler]
R A H U L

User avatar
GMAT Instructor
Posts: 3650
Joined: Wed Jan 21, 2009 4:27 am
Location: India
Thanked: 267 times
Followed by:80 members
GMAT Score:760

by sanju09 » Tue Dec 10, 2013 2:22 am
Patrick_GMATFix wrote:Unless I'm missing something, there is a problem with the question as you posted it. As written, the statements do not give us enough information, even when combined so the official answer should not be what you indicated.

Statement 1
Simplify (1) by dividing both sides by a. If a is positive, the inequality is maintained, and (1) becomes x>y. On the other hand, if a is negative, dividing by a will flip the sign, and (1) becomes x<y.
Because we can get conflicting results, statement (1) is NOT SUFFICIENT

Statement 2
We can think of a^2x as (a^2)^x. Likewise, a^2y = (a^2)^y. Thus, (2) can be rewritten as (a^2)^x > (a^2)^y. Now, just think of a^2 as an unknown positive value. If this value is greater than 1, for example if a^2 = 4, then (a^2)^x > (a^2)^y would mean that x>y. However, if this value is less than 1, for example if a^2=1/4, then (a^2)^x > (a^2)^y would mean that x<y (because as the exponent of a proper fraction increases, the result only gets smaller; for example, x=1, y=2 would work).
Because we can get conflicting results, statement (2) is NOT SUFFICIENT

Merge Statements
The quick way: we determined from (1) that x>y if a is positive, but x<y if a is negative. So for the statements together to be sufficient, (2) should give us a way to determine the sign of a. Since (2) gives us info on a^2, there is no way to isolate the sign of a. Thus the statements together are not sufficient..

The other quick way: we determined from (2) that x>y if a^2>1 (meaning if a>1 or <-1), but x<y if a^2<1 (meaning a is between -1 and 1). So for the statements together to be sufficient, (1) should give us a way to determine whether a falls between -1 and 1. Since (1) doesn't put any restriction on the value of a, we cannot determine this. Thus the statements together are not sufficient.

We can prove that the statements are not sufficient together by plugging in values that agree with both statements but still yield conflicting answers.

If a=2, x=2, y=1 both statements are respected, and x>y
If a=-1/2, x=1, y=2 both statements are respected, and x<y
We have conflicting results. The answer is E

If you correctly copied the question, beware of its source.

Hope this helps. Read my signature below for more
-Patrick
Your work in bold is misinterpretation of the situation presented. [spoiler]B[/spoiler] is the correct answer.
The mind is everything. What you think you become. -Lord Buddha



Sanjeev K Saxena
Quantitative Instructor
The Princeton Review - Manya Abroad
Lucknow-226001

www.manyagroup.com